Practice Exam Questions HEENT

Ace your homework & exams now with Quizwiz!

While discussing the history of a 6-month-old infant, the mother tells the nurse that she took a significant amount of aspirin while she was pregnant. What question would the nurse want to include in the history? a. "Does your baby seem to startle with loud noises?" b. "Has your baby had any surgeries on her ears?" c. "Have you noticed any drainage from her ears?" d. "How many ear infections has your baby had since birth?"

ANS: A Children exposed in utero to a variety of conditions, such as maternal rubella or to maternal ototoxic drugs are at risk for hearing deficits. Aspirin can be ototoxic, so the nurse should ask if the baby seems to startle with loud noises.

During an oral examination of a 4-year-old American-Indian child, the nurse notices that her uvula is partially split. Which of these statements is accurate? a. A bifid uvula may occur in some American-Indian groups. b. This condition is a cleft palate and is common in American Indians. c. A bifid uvula is torus palatinus, which frequently occurs in American Indians. d. This condition is due to an injury and should be reported to the authorities

ANS: A Bifid uvula, a condition in which the uvula is split either completely or partially, occurs in some American-Indian groups. This finding is not a cleft palate, a torus palatinus (benign bony ridge running in the middle of the hard palate), or due to injury.

A patient's laboratory data reveal an elevated thyroxine (T4) level. What gland should the nurse assess? a. Thyroid b. Parotid c. Adrenal d. Parathyroid

ANS: A The thyroid gland is a highly vascular endocrine gland that secretes T4 and triiodothyronine (T3). The other glands do not secrete T4. The parotid glands are salivary glands and secrete saliva. The adrenal glands secrete corticosteroids, not T4, and the parathyroid glands control the body's calcium. The gland that secretes thyroxine, or T4, is the thyroid gland. The thyroid gland is a highly vascular endocrine gland that secretes thyroxine (T4) and triiodothyronine (T3).

A 40-year-old patient who has just finished chemotherapy for breast cancer tells the nurse that she is concerned about her mouth. During the assessment the nurse finds areas of buccal mucosa that are raw and red with some bleeding, as well as other areas that have a white, cheesy coating. What do these findings indicate? a. Candidiasis b. Leukoplakia c. Koplik spots d. Aphthous ulcers

ANS: A Candidiasis is a white, cheesy, curdlike patch on the buccal mucosa and tongue. It scrapes off, leaving a raw, red surface that easily bleeds. It also occurs after the use of antibiotics or corticosteroids and in people who are immunosuppressed. Leukoplakia appears as chalky white, thick, raised patches with well-defined borders on the buccal mucosa. Koplik spots are small blue-white spots with irregular red halo scattered over mucosa opposite the molars and is an early sign of measles. Aphthous ulcers, or canker sores, first appear as a vesicle and then a small, round, "punched out" ulcer with a white base surrounded by a red halo and are quite painful and last for 1-2 weeks. The findings for this patient indicate candidiasis.

The nurse is performing an ear examination of an 80-year-old patient. Which of these findings would be considered normal? a. High-tone frequency loss b. Thin, translucent membrane c. Shiny, pink tympanic membrane d. Increased elasticity of the pinna

ANS: A A high-tone frequency hearing loss is apparent for those affected with presbycusis, the hearing loss that occurs with aging. The pinna loses elasticity (not increases), causing ear lobes to be pendulous. The eardrum may be whiter in color and more opaque and duller (not translucent or shiny, pink) in the older person than in the younger adult.

During an assessment of a 20-year-old patient with a 3-day history of nausea and vomiting, the nurse notices dry mucosa and deep vertical fissures in the tongue. What do these findings indicate? a. Dehydration b. A normal oral assessment c. Irritation from gastric juices d. Side effects from nausea medication

ANS: A Dehydration can cause dry mouth and deep vertical fissures in the tongue (due to reduced tongue volume). These finding are not normal and are not associated with irritation from gastric juices or from nausea caused by medications.

The nurse is preparing to do an otoscopic examination on a 2-year-old child. Which one of these reflects the correct procedure? a. Pulling the pinna down b. Pulling the pinna up and back c. Slightly tilting the child's head toward the examiner d. Instructing the child to touch their chin to their chest

ANS: A For an otoscopic examination on an infant or a child under 3 years of age, the pinna is pulled down. The other responses are not part of the correct procedure. The pinna should be pulled up and back for an otoscopic exam of an adult, not a child under 3 years of age. The child's head should be tilted slightly away from the examiner towards the opposite shoulder, not towards the examiner or to their chin. For an otoscopic examination on an infant or on a child under 3 years of age, the pinna is pulled down.

The nurse is using an otoscope to assess the nasal cavity. Which of these techniques is correct? a. Avoiding touching the nasal septum with the speculum b. Inserting the speculum at least 3 cm into the vestibule c. Gently displacing the nose to the side that is being examined d. Keeping the speculum tip medial to avoid touching the floor of the nares

ANS: A The correct technique for using an otoscope to examine the nasal cavity is to insert the apparatus into the nasal vestibule, avoiding pressure on the sensitive nasal septum. The tip of the nose should be lifted up before inserting the speculum.

When assessing the tongue of an adult, what finding would be considered abnormal? a. Smooth glossy dorsal surface b. Thin white coating over the tongue c. Raised papillae on the dorsal surface d. Visible venous patterns on the ventral surface

ANS: A The dorsal surface of the tongue is normally roughened from papillae. A thin white coating may be present. The ventral surface may show veins. Smooth, glossy areas are abnormal and may indicate atrophic glossitis.

The nurse is performing an assessment on a 7-year-old child who has symptoms of chronic watery eyes, sneezing, and clear nasal drainage. The nurse notices the presence of a transverse line across the bridge of the nose, dark blue shadows below the eyes, and a double crease on the lower eyelids. What does the nurse suspect is the cause of these signs and symptoms? a. Chronic allergies b. Lymphadenopathy c. Nasal congestion d. Upper respiratory infection

ANS: A Chronic allergies often develop chronic facial characteristics and include blue shadows below the eyes, a double or single crease on the lower eyelids, open-mouth breathing, and a transverse line on the nose. Lymphadenopathy means enlargement of lymph nodes from infection, allergy, or neoplasm, it is a sign of allergies, not a cause. Although nasal congestion and upper respiratory infections may present with watery eyes and sneezing, people with nasal congestion usually state congestion or a pressure feeling in their head and people with upper respiratory infections often have a cough and/or sore throat and don't have a transverse line across the bridge of the nose, dark blue shadows under the eyes, or a double crease on the lower eyelids. The signs and symptoms of this patient are likely from chronic allergies.

During a well-baby checkup, a mother is concerned because her 2-month-old infant cannot hold her head up when she is pulled to a sitting position. Which response by the nurse is appropriate? a. "Head control is usually achieved by 4 months of age." b. "You shouldn't be trying to pull your baby up like that until she is older." c. "Head control should be achieved by this time." d. "This inability indicates possible nerve damage to the neck muscles."

ANS: A Head control is achieved by 4 months when the baby can hold the head erect and steady when pulled to a vertical position. The other responses are not appropriate.

A 2-week-old infant can fixate on an object but cannot follow a light or bright toy. What should the nurse do? a. Consider this a normal finding. b. Continue with the examination, and assess visual fields. c. Assess the pupillary light reflex for possible blindness. d. Expect that a 2-week-old infant should be able to fixate and follow an object.

ANS: A This is a normal finding. By 2 to 4 weeks an infant can fixate on an object. By the age of 1 month, the infant should fixate and follow a bright light or toy. This is a normal finding. The nurse cannot assess visual fields as the infant cannot follow instructions; assessing the pupillary light reflex does not assess for blindness; and the nurse should not expect the infant to be able follow an object. An infant can fixate on an object by 2 to 4 weeks and by the age of 1 month, should be able to fixate and follow a bright light or toy.

A 68-year-old woman is in the eye clinic for a checkup. She tells the nurse that she has been having trouble reading the paper, sewing, and even seeing the faces of her grandchildren. On examination, the nurse notes that she has some loss of central vision but her peripheral vision is normal. What do these findings suggest? a. Macular degeneration b. Vision that is normal for someone her age c. The beginning stages of cataract formation d. Increased intraocular pressure or glaucoma

ANS: A Macular degeneration is characterized by the loss of central vision and is the most common cause of blindness. Cataracts would show lens opacity. Chronic open-angle glaucoma, the most common type of glaucoma, involves a gradual loss of peripheral vision. These findings are not consistent with normal vision at this, or any, age. These findings are not consistent with normal vision at this, or any, age. The increased intraocular pressure of chronic open-angle glaucoma, the most common type of glaucoma, involves a gradual loss of peripheral vision but not central vision. The symptoms this patient has suggest macular degeneration. Macular degeneration is characterized by the loss of central vision and is the most common cause of blindness

When performing the corneal light reflex assessment, the nurse notes that the light is reflected at 2 o'clock in each eye. What should the nurse do regarding this finding? a. Record this as a normal finding. b. Refer the individual for further evaluation. c. Document this finding as an asymmetric light reflex. d. Perform the confrontation test to validate the findings.

ANS: A Reflection of the light on the corneas should be in exactly the same spot on each eye, or symmetric. If asymmetry is noted, then the nurse should administer the cover test.

A physician tells the nurse that a patient's vertebra prominens is tender and asks the nurse to reevaluate the area in 1 hour. What area of the body will the nurse assess? a. At the level of the C7 vertebra b. At the level of the T11 vertebra c. At the level of the L5 vertebra d. At the level of the S3 vertebra

ANS: A The C7 vertebra has a long spinous process, called the vertebra prominens, which is palpable when the head is flexed.

The nurse is performing an external eye examination. Which statement regarding the outer layer of the eye is true? a. The outer layer of the eye is very sensitive to touch. b. The outer layer of the eye is darkly pigmented to prevent light from reflecting internally. c. The trigeminal nerve (CN V) and the trochlear nerve (CN IV) are stimulated when the outer surface of the eye is stimulated. d. The visual receptive layer of the eye in which light waves are changed into nerve impulses is located in the outer layer of the eye.

ANS: A The cornea and the sclera make up the outer layer of the eye. The cornea is very sensitive to touch. The middle layer, the choroid, has dark pigmentation to prevent light from reflecting internally. The trigeminal nerve (CN V) and the facial nerve (CN VII), not the trochlear nerve (IV), are stimulated when the outer surface of the eye is stimulated. The retina, in the inner layer of the eye, is where light waves are changed into nerve impulses. The outer layer of the eye is not darkly pigmented, the sclera is white and the cornea is transparent. It is the middle layer, the choroid, that has dark pigmentation to prevent light from reflecting internally. The true statement about the outlayer of the eye is that it is made up of the cornea and the sclera.

The nurse is reviewing the age-related changes in the eye for a class. Which of these physiologic changes is responsible for presbyopia? a. Loss of lens elasticity b. Degeneration of the cornea c. Decreased adaptation to darkness d. Decreased distance vision abilities

ANS: A The lens loses elasticity and decreases its ability to change shape to accommodate for near vision. This condition is called presbyopia.

When examining a patient's CN function, what muscles should the nurse assess to assess the function of CN XI? a. Sternomastoid and trapezius b. Spinal accessory and omohyoid c. Trapezius and sternomandibular d. Sternomandibular and spinal accessory

ANS: A The muscles innervated by CN XI are the sternomastoid and the trapezius muscles in the neck. Options B, C, and D are incorrect because the spinal accessory is not a muscle but the name of CN XI and there is no sternomandibular muscle

The nurse is assessing a patient who may have hearing loss. Which of these statements is true concerning air conduction? a. Air conduction is the normal pathway for hearing. b. Amplitude of sound determines the pitch that is heard. c. Vibrations of the bones in the skull cause air conduction. d. Loss of air conduction is called a conductive hearing loss.

ANS: A The normal pathway of hearing is air conduction, which starts when sound waves produce vibrations on the tympanic membrane. Conductive hearing loss results from a mechanical dysfunction of the external or middle ear. The other statements are not true concerning air conduction. The frequency of sound waves is what determines pitch, not the amplitude. Vibrations of the bones in the skull are bone conduction, not air conduction. Conductive hearing loss involves mechanical dysfunction of the external or middle ear and is caused by impacted cerumen, foreign bodies, a perforated tympanic membrane, pus or serum in the middle ear, and otosclerosis, not loss of air conduction.

The nurse is examining a patient's retina with an ophthalmoscope. Which finding is considered normal? a. Optic disc that is a yellow-orange color. b. Presence of pigmented crescents in the macular area. c. Optic disc margins that are blurred around the edges. d. Presence of the macula located on the nasal side of the retina

ANS: A The optic disc is located on the nasal side of the retina. Its color is a creamy yellow-orange to a pink, and the edges are distinct and sharply demarcated, not blurred. A pigmented crescent is black and is due to the accumulation of pigment in the choroid. Presence of pigmented crescents in the macular area is an abnormal finding. A pigmented crescent is black and is due to the accumulation of pigment in the choroid. The optic dish margins are normally distinct and sharply demarcated, not blurred around the edges. The macula is located on the temporal side of the fundus of the eye, not on the nasal side of the retina. The correct answer of a normal finding is that the optic disc is a yellow-orange color.

When examining the eye, the nurse notices that the patient's eyelid margins approximate completely. What does the nurse understand about this assessment finding? a. It is expected. b. It may result in problems with tearing. c. It indicates increased intraocular pressure. d. It may indicate a problem with extraocular muscles

ANS: A The palpebral fissure is the elliptical open space between the eyelids, and, when closed, the lid margins approximate completely, which is a normal finding. This is a normal finding and does not result in problems with tearing or indicate problems with increase intraocular pressure or extraocular muscles

During an examination, the nurse knows that the best way to palpate the lymph nodes in the neck is described by which statement? a. Using gentle pressure, palpate with both hands to compare the two sides. b. Using strong pressure, palpate with both hands to compare the two sides. c. Gently pinch each node between one's thumb and forefinger, and then move down the neck muscle. d. Using the index and middle fingers, gently palpate by applying pressure in a rotating pattern.

ANS: A Using gentle pressure is recommended because strong pressure can push the nodes into the neck muscles. Palpating with both hands to compare the two sides symmetrically is usually most efficient.

When examining children affected with Down syndrome (trisomy 21), what should the nurse look for r/t this disorder? a. Ear dysplasia b. Long, thin neck c. Protruding thin tongue d. Narrow and raised nasal bridge

ANS: A With the chromosomal aberration trisomy 21, also known as Down syndrome, head and face characteristics may include upslanting eyes with inner epicanthal folds, a flat nasal bridge, a small broad flat nose, a protruding thick tongue, ear dysplasia, a short broad neck with webbing, and small hands with a single palmar crease. A short broad neck with webbing, rather than a long, thin neck; a protruding thick tongue, not a thin tongue; and a flat nasal bridge, not a raised nasal bridge are associated with Down syndrome.

The nurse suspects that a patient has hyperthyroidism, and the laboratory data indicate that the patient's T4 and T3 hormone levels are elevated. Which of these findings would the nurse most likely find on examination? a. Dyspnea b. Tachycardia c. Constipation d. Atrophied nodular thyroid gland

ANS: B T4 and T3 are thyroid hormones that stimulate the rate of cellular metabolism, resulting in tachycardia. With an enlarged thyroid gland as in hyperthyroidism, the nurse might expect to find diffuse enlargement (goiter) or a nodular lump but not an atrophied gland. Dyspnea and constipation are not findings associated with hyperthyroidism.

In performing a voice test to assess hearing, which of these actions would the nurse perform? a. Shield the lips so that the sound is muffled. b. Whisper a set of random numbers and letters, and then ask the patient to repeat them. c. Ask the patient to place his or her finger in their ear to occlude outside noise. d. Stand approximately 4 feet away to ensure that the patient can really hear at this distance.

ANS: B With the examiner's head 30 to 60 cm (1 to 2 feet) from the patient's ear, have the patient place one finger on the tragus of the ear and push it in and out of the auditory meatus. While the patient is doing this, the examiner exhales and slowly whispers a set of random numbers and letters, such as "5, B, 6." Normally the patient is asked to repeat each number and letter correctly after hearing the examiner say them. Shielding the lips to muffle the sound, asking the patient to place a finger in their ear to occlude outside noise, and the examiner standing 4 feet away from the patient are not techniques used to perform the voice test. The voice test is performed with the examiner's head 30 to 60 cm (1 to 2 feet) from the patient's ear and having the patient place one finger on the tragus of the ear and push it in and out of the auditory meatus. While the patient is doing this, the examiner exhales and slowly whispers a set of random numbers and letters, such as "5, B, 6." Normally, the patient is asked to repeat each number and letter correctly after hearing the examiner say them and then exhaling and slowly whispering a set of random numbers and letters, such as "5, B, 6." Normally the patient is asked to repeat each number and letter correctly after hearing the examiner say them.

When performing an otoscopic examination of a 5-year-old child with a history of chronic ear infections, the nurse sees that his right tympanic membrane is amber-yellow in color and that air bubbles are visible behind the tympanic membrane. The child reports occasional hearing loss and a popping sound with swallowing. Based on this data, what does the nurse conclude? a. An acute purulent otitis externa b. Most likely a serous otitis media c. Evidence of a resolving cholesteatoma d. Experiencing the early stages of perforation

ANS: B An amber-yellow color to the tympanic membrane suggests serum or pus in the middle ear. Air or fluid or bubbles behind the tympanic membrane are often visible. The patient may have feelings of fullness, transient hearing loss, and a popping sound with swallowing. These findings most likely suggest that the child has serous otitis media. The other responses are not correct. The manifestation of otitis externa is a sticky, yellow discharge (not an amber-yellow tympanic membrane). Cholesteatoma is an overgrowth of epidermal tissue in the middle ear or temporal bone that has a pearly white, cheesy appearance (not an amber-yellow color). A perforation typically begins with ear pain and stops with a popping sensation and then drainage occurs. This patient's amber-yellow color to the tympanic membrane suggests serum or pus in the middle ear. Air or fluid or bubbles behind the tympanic membrane are often visible.

A patient has been admitted after an accident at work. During the assessment, the patient is having trouble hearing and states, "I don't know what is the matter. All of a sudden I can't hear you out of my left ear!" What should the nurse do next? a. Irrigate the ear with rubbing alcohol. b. Notify the patient's health care provider. c. Prepare to remove cerumen from the patient's ear. d. Make note of this finding for the report to the next shift.

ANS: B Any sudden loss of hearing in one or both ears that is not associated with an upper respiratory infection needs to be reported at once to the patient's health care provider. Hearing loss associated with trauma is often sudden. Irrigating the ear or removing cerumen is not appropriate at this time. This is not a normal finding.

The nurse is performing a middle ear assessment on a 15-year-old patient who has had a history of chronic ear infections. When examining the right tympanic membrane, the nurse sees the presence of dense white patches. The tympanic membrane is otherwise unremarkable. It is pearly, with the light reflex at 5 o'clock and landmarks visible. What should the nurse do? a. Refer the patient for the possibility of a fungal infection. b. Recognize that these are scars caused from frequent ear infections. c. Consider that these findings may represent the presence of blood in the middle ear. d. Be concerned about the ability to hear because of this abnormality on the tympanic membrane.

ANS: B Dense white patches on the tympanic membrane are sequelae of repeated ear infections. They do not necessarily affect hearing. A fungal infection manifests as a colony of black or white dots on the eardrum or canal walls (not dense white patch). Blood behind the tympanic membrane would cause the tympanic membrane to appear blue or dark red

When examining the mouth of an older patient, the nurse recognizes which finding is due to the aging process? a. Teeth appearing shorter b. Tongue that looks smoother in appearance c. Buccal mucosa that is beefy red in appearance d. Small, painless lump on the dorsum of the tongue

ANS: B In the aging adult, the tongue looks smoother because of papillary atrophy. The teeth are slightly yellowed and appear longer because of the recession of gingival margins.

In assessing the tonsils of a 30-year-old, the nurse notices that they are involuted, granular in appearance, and appear to have deep crypts. What is the correct response to these findings? a. Refer the patient to a throat specialist. b. No response is needed; this appearance is normal for the tonsils. c. Continue with the assessment, looking for any other abnormal findings. d. Obtain a throat culture on the patient for possible streptococcal (strep) infection.

ANS: B The tonsils are the same color as the surrounding mucous membrane, although they look more granular and their surface shows deep crypts. Tonsillar tissue enlarges during childhood until puberty and then involutes. There is no need to refer the patient to a throat specialist, obtain a throat culture, or look for other abnormal findings because the findings in this question are normal. Although the tonsils look more granular and their surface shows deep crypts, they are the same color as the surrounding mucous membrane and tonsillar tissue enlarges during childhood until puberty and then involutes.

A patient in her first trimester of pregnancy is diagnosed with rubella. Which of these statements is correct regarding the significance of this in relation to the infant's hearing? a. Rubella may affect the mother's hearing but not the infant's. b. Rubella can damage the infant's organ of Corti, which will impair hearing. c. Rubella can impair the development of cranial nerve VIII and thus affect hearing. d. Rubella is especially dangerous to the infant's hearing in the second trimester of pregnancy

ANS: B If maternal rubella infection occurs during the first trimester, then it can damage the organ of Corti and impair hearing. Maternal rubella can affect the infant's hearing, not the mother's hearing, if it occurs in the first trimester of pregnancy. Hearing is impaired due to damage to the organ of Corti, not cranial nerve VIII. Rubella does not impair the development of cranial nerve VIII.

When examining the ear with an otoscope, how should the tympanic membrane look? a. Light pink with a slight bulge b. Pearly gray and slightly concave c. Whitish with black flecks or dots d. Pulled in at the base of the cone of light

ANS: B The tympanic membrane is a translucent membrane with a pearly gray color and a prominent cone of light in the anteroinferior quadrant, which is the reflection of the otoscope light. The tympanic membrane is oval and slightly concave, pulled in at its center by the malleus, which is one of the middle ear ossicles. A light pink color and a slight bulge of the tympanic membrane indicate otitis media. It should not look white and if there are tiny black flecks or dots, that is indicative of a fungal infection, or otomycosis. The tympanic membrane does not appear pulled in at the base of the cone of light, but should instead appear flat and slightly pulled in at the center. A normal tympanic membrane should appear a pearly gray color and have a prominent cone of light in the anteroinferior quadrant, which is the reflection of the otoscope light. The tympanic membrane is oval and slightly concave, pulled in at its center by the malleus, which is one of the middle ear ossicles

A 10-year-old is at the clinic for "a sore throat that has lasted 6 days." Which of these findings would be consistent with an acute infection? a. Tonsils 3+/1-4+ with pale coloring b. Tonsils 3+/1-4+ with large white spots c. Tonsils 2+/1-4+ with small plugs of white debris d. Tonsils 1+/1-4+ and pink; the same color as the oral mucosa

ANS: B With an acute infection, tonsils are bright red and swollen and may have exudate or large white spots. Tonsils are enlarged to 2+, 3+, or 4+ with an acute infection.

A 17-year-old student is a swimmer on her high school's swim team. She has had three bouts of otitis externa this season and wants to know what to do to prevent it. What should the nurse include in the instructions? a. Use a cotton-tipped swab to dry ear canals thoroughly after each swim. b. Use rubbing alcohol or 2% acetic acid eardrops after every swim. c. Irrigate the ears with warm water and a bulb syringe after each swim. d. Rinse the ears with a warmed solution of mineral oil and hydrogen peroxide.

ANS: B With otitis externa (swimmer's ear), swimming causes the external canal to become waterlogged and swell; skinfolds are set up for infection. The rubbing alcohol and acetic acid mix with the water in the ear and then evaporate. The use of cotton-tip swabs in the ears is not recommended as cotton can be left in the ear and it can also impact cerumen. Irrigating the ears is done to clean the ears, not prevent otitis externa. Otitis externa can be prevented by using rubbing alcohol or 2% acetic acid eardrops after every swim.

A patient comes into the clinic reporting pain in her O.D. On examination, the nurse sees a pustule at the lid margin that is painful to touch, red, and swollen. What is the correct term for this finding? a. Chalazion b. Hordeolum c. Blepharitis d. Dacryocystitis

ANS: B A hordeolum, or stye, is a painful, red, and swollen pustule at the lid margin. A chalazion is a nodule protruding on the lid, toward the inside, and is nontender, firm, with discrete swelling. Dacryocystitis is an inflammation of the lacrimal sac. Blepharitis is inflammation of the eyelids. The painful, red, swollen pustule at the lid margin on this patient's eyelid is a hordeolum or stye.

The nurse is performing the diagnostic positions test. Which result is a normal finding? a. Convergence of the eyes b. Parallel movement of both eyes c. Nystagmus in extreme superior gaze d. Slight amount of lid lag when moving the eyes from a superior to an inferior position

ANS: B A normal response for the diagnostic positions test is parallel tracking of the object with both eyes. Eye movement that is not parallel indicates a weakness of an extraocular muscle or dysfunction of the CN that innervates it. The diagnostic positions test assesses for any muscle weakness during movement of the eye by leading the eyes through the six cardinal positions of gaze. It is not assessing the ability of the eyes to converge, or move toward each other. Nystagmus with an extreme superior gaze is normal, but not in any other position and lid lag is not normal.

Which of these assessment findings would the nurse expect to see when examining the eyes of a black patient? a. Increased night vision b. Dark retinal background c. Increased photosensitivity d. Narrowed palpebral fissures

ANS: B An ethnically based variability in the color of the iris and in retinal pigmentation exists, with darker irides having darker retinas behind them.

A patient reports excruciating headache pain on one side of his head, especially around his eye, forehead, and cheek that has lasted approximately 1 2 to 2 hours, occurring once or twice each day. What should the nurse suspect? a. Hypertension b. Cluster headaches c. Tension headaches d. Migraine headaches

ANS: B Cluster headaches produce pain around the eye, temple, forehead, and cheek and are unilateral and always on the same side of the head. They are excruciating and occur once or twice per day and last 1 2 to 2 hours each. Although hypertension may cause headaches, the blood pressure needs to be severely elevated and would likely not occur once or twice a day and last for 1 2 to 2 hours. Tension headaches are occipital, frontal, or with bandlike tightness. Migraine headaches are supraorbital, retro-orbital, or frontotemporal.

During an assessment of an infant, the nurse notes that the fontanels are depressed and sunken. The nurse suspects which condition? a. Rickets b. Dehydration c. Mental retardation d. Increased intracranial pressure

ANS: B Depressed and sunken fontanels occur with dehydration or malnutrition. Mental retardation and rickets have no effect on the fontanels. Increased intracranial pressure would cause tense or bulging and possibly pulsating fontanels

The nurse notices that a patient's palpebral fissures are not symmetric. On examination, the nurse may find that damage has occurred to which cranial nerve (CN)? a. V b. VII c. XI d. XIII

ANS: B Facial muscles are mediated by CN VII; asymmetry of palpebral fissures may be attributable to damage to CN VII (Bell palsy). Cranial nerve V, the trigeminal nerve, mediates facial sensations of pain and touch. Cranial nerve XI is the spinal accessory nerve that innervates the sternomastoid and trapezius muscles of the neck. There is no cranial nerve XIII (only 12 cranial nerves).

A patient has come in for an examination and states, "I have this spot in front of my ear lobe on my cheek that seems to be getting bigger and is tender. What do you think it is?" The nurse notes swelling below the angle of the jaw. What does the nurse suspect? a. Inflammation of the thyroid gland b. Inflammation of the parotid gland c. Infection in the occipital lymph node d. Infection in the submental lymph node

ANS: B Swelling of the parotid gland is evident below the angle of the jaw and is most visible when the head is extended. Painful inflammation occurs with mumps, and swelling also occurs with abscesses or tumors. Swelling occurs anterior to the lower ear lobe.

A patient's thyroid gland is enlarged, and the nurse is preparing to auscultate the thyroid gland for the presence of a bruit. What technique should the nurse use to assess for a bruit. a. Palpate the thyroid while the patient is swallowing. b. Auscultate the thyroid with the bell of the stethoscope. c. Palpate the thyroid while the patient holds their breath. d. Auscultate the thyroid with the diaphragm of the stethoscope.

ANS: B If the thyroid gland is enlarged, then the nurse should auscultate it for the presence of a bruit, which is a soft, pulsatile, whooshing, blowing sound heard best with the bell of the stethoscope. A bruit is a soft, pulsatile, whooshing, blowing sound. A bruit occurs with accelerated or turbulent blood flow. It is not able to be palpated. A bruit is heard best with the bell, not the diaphragm of the stethoscope

The nurse has just completed a lymph node assessment on a 60-year-old healthy female patient. When palpating the nodes on this healthy 60-year-old adult, how did the lymph nodes feel? a. Fixed b. Nonpalpable c. Rubbery, discrete, and mobile d. Large, firm, and fixed to the tissue

ANS: B Most lymph nodes are nonpalpable in adults. The palpability of lymph nodes decreases with age. Normal nodes feel movable, discrete, soft, and nontender.

During ocular examinations, what should the nurse keep in mind regarding the movement of the extraocular muscles? a. Is decreased in the older adult. b. Is stimulated by CNs III, IV, and VI. c. Is impaired in a patient with cataracts. d. Is stimulated by cranial nerves (CNs) I and II.

ANS: B Movement of the extraocular muscles is stimulated by three CNs: III, IV, and VI. Aging and cataracts do not affect the extraocular movements. Movement of the extraocular muscles is not stimulated by CNs I and II but by CNs III, IV, and VI.

A woman comes to the clinic and states, "I've been sick for so long! My eyes have gotten so puffy, and my eyebrows and hair have become coarse and dry." For what condition should the nurse assess for other signs and symptoms? a. Cachexia b. Myxedema c. Graves disease d. Parkinson syndrome

ANS: B Myxedema (hypothyroidism) is a deficiency of thyroid hormone that, when severe, causes a nonpitting edema or myxedema. The patient will have a puffy edematous face, especially around the eyes (periorbital edema); coarse facial features; dry skin; and dry, coarse hair and eyebrows. Cachexia, or cachectic appearance, accompanies chronic wasting diseases such as cancer, dehydration, and starvation. Features included sunken eyes, hollow cheeks, and exhausted, defeated expression. Graves disease is an autoimmune disease with increased production of thyroid hormones which is manifested by goiter, eyelid retraction, and exophthalmos (bulging eyeballs) and other symptoms. The facial features characteristic of Parkinson syndrome are a flat and expressionless, "masklike," with elevated eyebrows, staring gaze, oily skin, and drooling. The signs and symptoms of this patient are characteristic of myxedema, or hypothyroidism.

A patient's vision is recorded as 20/80 in each eye. How does the nurse interpret this finding? a. Patient has presbyopia. b. Patient as poor vision. c. Patient has acute vision. d. Patient has normal vision.

ANS: B Normal visual acuity is 20/20 in each eye; the larger the denominator, the poorer the vision. Presbyopia is a decrease in accommodation which is observed by convergence (motion toward) of the axes of the eyeballs and pupillary constriction and is tested by having the person focus on a distant object.

During an assessment of the sclera of a black patient, the nurse would consider which of these an expected finding? a. Yellow fatty deposits over the cornea b. Presence of small brown macules on the sclera c. Pallor near the outer canthus of the lower lid d. Yellow color of the sclera that extends up to the iris

ANS: B Normally in dark-skinned people, small brown macules may be observed in the sclera. Blacks may have yellowish fatty deposits beneath the eyelids, away from the cornea, not over the cornea or extending up to the iris. Pallor near the outer canthus is not normal, but may indicate anemia.

A 31-year-old patient tells the nurse that he has noticed a progressive loss in his hearing. He says that it does seem to help when people speak louder or if he turns up the volume of a television or radio. What is the most likely cause of this hearing loss? a. Presbycusis b. Otosclerosis c. Trauma to the bones d. Frequent ear infections

ANS: B Otosclerosis is a common cause of conductive hearing loss in young adults between the ages of 20 and 40 years. Presbycusis is a type of hearing loss that occurs with aging. Trauma and frequent ear infections are not a likely cause of his hearing loss. Instead, a common cause of conductive hearing loss (mechanical dysfunction of the external or middle ear which causes partial hearing loss that can be compensated for with an increase in amplitude) in young adults between the ages of 20 and 40 years is otosclerosis.

A patient says that she has recently noticed a lump in the front of her neck below her "Adam's apple" that seems to be getting bigger. During the assessment, what finding would lead the nurse to suspect that this may not be a cancerous thyroid nodule? a. It is tender. b. It is mobile and soft. c. It disappears when the patient smiles. d. It is hard and fixed to the surrounding structures.

ANS: B Painless, rapidly growing nodules may be cancerous, especially the appearance of a single nodule in a young person. However, cancerous nodules tend to be hard and fixed to surrounding structures, not mobile.

The nurse is performing an eye assessment on an 80-year-old patient. Which of these findings is considered abnormal? a. Decrease in tear production b. Unequal pupillary constriction in response to light c. Presence of arcus senilis observed around the cornea d. Loss of the outer hair on the eyebrows attributable to a decrease in hair follicles

ANS: B Pupils are small in the older adult, and the pupillary light reflex may be slowed, but pupillary constriction should be symmetric. The assessment findings in the other responses are considered normal in older people. Decrease in tear production, presence of arcus senilis around the cornea, and loss of outer hair on the eyebrows are normal findings in the aging adult. Although pupils are small in the older adult, and the pupillary light reflex may be slowed, the pupillary constriction should be symmetric. Therefore, unequal pupillary constriction in response to light is an abnormal finding.

A mother is concerned because her 18-month-old toddler has 12 teeth. She is wondering if this is normal for a child of this age. Which is the best response by the nurse? a. "How many teeth did you have at this age?" b. "This is a normal number of teeth for an 18 month old." c. "Normally, by age 2 1/2 years, 16 deciduous teeth are expected." d. "All 20 deciduous teeth are expected to erupt by age 4 years."

ANS: B The guidelines for the number of teeth for children younger than 2 years old are as follows: the child's age in months minus the number 6 should be equal to the expected number of deciduous teeth. Normally all 20 teeth are in by 21 2 years old. In this instance, the child is 18 months old, minus 6, equals 12 deciduous teeth expected.

The nurse is conducting a visual examination. Which of these statements regarding visual pathways and visual fields is true? a. The right side of the brain interprets the vision for the O.D. b. The image formed on the retina is upside down and reversed from its actual appearance in the outside world. c. Light rays are refracted through the transparent media of the eye before striking the pupil. d. Light impulses are conducted through the optic nerve to the temporal lobes of the brain.

ANS: B The image formed on the retina is upside down and reversed from its actual appearance in the outside world. The light rays are refracted through the transparent media of the eye before striking the retina, and the nerve impulses are conducted through the optic nerve tract to the visual cortex of the occipital lobe of the brain. The left side of the brain interprets vision for the O.D.

A mother brings her newborn in for an assessment and asks, "Is there something wrong with my baby? His head seems so big." Which statement is true regarding the relative proportions of the head and trunk of the newborn? a. At birth, the head is one fifth the total length. b. Head circumference should be greater than chest circumference at birth. c. The head size reaches 90% of its final size when the child is 3 years old. d. When the anterior fontanel closes at 2 months, the head will be more proportioned to the body.

ANS: B The nurse recognizes that during the fetal period, head growth predominates. Head size is greater than chest circumference at birth, and the head size grows during childhood, reaching 90% of its final size when the child is age 6 years

A male patient with a history of acquired immunodeficiency syndrome (AIDS) has come in for an examination and states, "I think that I have the mumps." What should the nurse examine first? a. Thyroid gland b. Parotid gland c. Cervical lymph nodes d. Mouth and skin for lesions

ANS: B The parotid gland may become swollen with the onset of mumps, and parotid enlargement has been found with HIV

The nurse is palpating the sinus areas. If the findings are normal, then the patient should report which sensation? a. No sensation b. Firm pressure c. Pain during palpation d. Pain sensation behind eyes

ANS: B The person should feel firm pressure but no pain. Sinus areas are tender to palpation in persons with chronic allergies or an acute infection (sinusitis). A normal finding when palpating the sinus areas is for the patient to feel firm pressure, not no sensation at all, pain during palpation, or pain behind the eyes. Sinus areas that are tender to palpation may indicate chronic allergies or an acute infection (sinusitis). Feeling firm pressure but no pain is a normal finding.

A patient's vision is recorded as 20/30 when the Snellen eye chart is used. How should the nurse interpret these results? a. At 30 feet the patient can read the entire chart. b. The patient can read at 20 feet what a person with normal vision can read at 30 feet. c. The patient can read the chart from 20 feet in the O.S. and 30 feet in the O.D. d. The patient can read from 30 feet what a person with normal vision can read from 20 feet.

ANS: B The top number indicates the distance the person is standing from the chart; the denominator gives the distance at which a normal eye can see.

When examining the face of a patient, what are the two pairs of salivary glands that are accessible for examination? a. Occipital; submental b. Parotid; submandibular c. Submandibular; occipital d. Sublingual; parotid

ANS: B The two pairs of salivary glands accessible to examination on the face are the parotid glands, which are in the cheeks over the mandible, anterior to and below the ear; and the submandibular glands, which are beneath the mandible at the angle of the jaw. The parotid glands are normally nonpalpable. The occipital and submental are lymph nodes, not glands and the sublingual glands lie on the floor of the mouth, so are not readily accessible for examination. The two pairs of salivary glands that are accessible for examination on the face are the parotid glands, which are in the cheeks over the mandible, anterior to and below the ear; and the submandibular glands, which are beneath the mandible at the angle of the jaw.

While obtaining a health history, a patient tells the nurse that he has frequent nosebleeds and asks the best way to get them to stop. What would be the nurse's best response? a. "While sitting up, place a cold compress over your nose." b. "Sit up with your head tilted forward and pinch your nose." c. "Allow the bleeding to stop on its own, but don't blow your nose." d. "Lie on your back with your head tilted back and pinch your nose."

ANS: B With a nosebleed, the person should sit up with the head tilted forward and pinch the nose between the thumb and forefinger for 5 to 15 minutes.

A patient has been diagnosed with strep throat. The nurse is aware that without treatment, which complication may occur? a. Rubella b. Leukoplakia c. Scarlet fever d. Rheumatic fever

ANS: D Untreated strep throat may lead to rheumatic fever. When performing a health history, the patient should be asked whether his or her sore throat has been documented as streptococcal.

A patient is unable to differentiate between sharp and dull stimulation to both sides of her face. What does the nurse suspect? a. Bell palsy b. Scleroderma c. Damage to the trigeminal nerve d. Frostbite with resultant paresthesia to the cheeks

ANS: C Facial sensations of pain or touch are mediated by CN V, which is the trigeminal nerve. Bell palsy is associated with CN VII damage. Frostbite and scleroderma are not associated with this problem.

The nurse is assessing a 16-year-old patient who has suffered head injuries from a recent motor vehicle accident. Which of these statements indicates the most important reason for assessing for any drainage from the ear canal? a. The auditory canal may be occluded from increased cerumen. b. If the drum has ruptured, then purulent drainage will result. c. Bloody or clear watery drainage can indicate a basal skull fracture. d. Foreign bodies from the accident may cause occlusion of the ear canal.

ANS: C Frank blood or clear watery drainage (cerebrospinal fluid) after a trauma suggests a basal skull fracture and warrants immediate referral. Purulent drainage indicates otitis externa or otitis media. An ear canal occluded from cerumen would not be draining, purulent drainage indicates otitis externa or otitis media, and it is not likely a foreign body from an accident would cause occlusion of the ear canal.

The nurse is assessing a patient in the hospital who has received numerous antibiotics for a lung infection and notices that his tongue appears to be black and hairy. In response to his concern, what would the nurse say? a. "We will need to get a biopsy to determine the cause." b. "This is an overgrowth of hair and will go away in a few days." c. "Black, hairy tongue is a fungal infection caused by all the antibiotics you have received." d. "This is probably caused by the same bacteria you had in your lungs."

ANS: C A black, hairy tongue is not really hair but the elongation of filiform papillae and painless overgrowth of mycelial threads of fungus infection on the tongue. It occurs after the use of antibiotics, which inhibit normal bacteria and allow a proliferation of fungus. It is not caused by the same bacteria as his lung infection but occurred after the use of antibiotics, which inhibit normal bacteria and allow a proliferation of fungus. There is no need to get a biopsy.

The nurse is performing an assessment on a 65-year-old man. He reports a crusty nodule behind the pinna. It intermittently bleeds and has not healed over the past 6 months. On physical assessment, the nurse finds an ulcerated crusted nodule with an indurated base. Based on these findings, what does the nurse suspect? a. Most likely a keloid b. Probably a benign sebaceous cyst c. Could be a potential carcinoma, and the patient should be referred for a biopsy d. A tophus, which is common in the older adult and is a sign of gout

ANS: C An ulcerated crusted nodule with an indurated base that fails to heal is characteristic of a carcinoma. These lesions fail to heal and intermittently bleed. Individuals with such symptoms should be referred for a biopsy. The other responses are not correct. A keloid is an overgrowth of scar tissue which in the ear is common at lobule at the site of a pierced ear. A sebaceous cyst is a nodule filled with waxy sebaceous material, is painful if it becomes infected, and is often multiple of them. A tophus is a hard uric acid deposit under the skin. The ulcerated crusted nodule with an indurated base that fails to heal that this patient has is characteristic of a carcinoma.

During an assessment of a 20-year-old Asian patient, the nurse notices that he has dry, flaky, white cerumen in his canal. What is the significance of this finding? a. It represents poor hygiene. b. It is probably the result of lesions from eczema in his ear. c. It is a normal finding, and no further follow-up is necessary. d. It could be indicative of change in cilia; the nurse should assess for hearing loss.

ANS: C Asians and American Indians are more likely to have dry cerumen, which appears white and flaky, whereas blacks and whites usually have wet cerumen that appears honey-brown. Dry, flaky cerumen in an Asian patient is not a result of poor hygiene, lesions from eczema, or change in cilia.

The nurse is assessing a 3-year-old for "drainage from the nose." On assessment, a purulent drainage that has a very foul odor is noted from the left naris and no drainage is observed from the right naris. The child is afebrile with no other symptoms. What should the nurse do next? a. Refer to the physician for an antibiotic order. b. Have the mother bring the child back in 1 week. c. Perform an otoscopic examination of the left nares. d. Tell the mother that this drainage is normal for a child of this age.

ANS: C Children are prone to put an object up the nose, producing unilateral purulent drainage with a foul odor. Because some risk for aspiration exists, removal should be prompt.

The nurse is preparing to perform an otoscopic examination of a newborn infant. Which statement is true regarding this examination? a. An injected membrane may indicate an infection. b. The eardrum will appear in the oblique position. c. The normal membrane may appear thick and opaque. d. The appearance of the membrane is identical to that of an adult

ANS: C During the first few days after the birth, the tympanic membrane of a newborn often appears thickened and opaque. It may look injected and have a mild redness from increased vascularity. The eardrum of a neonated is more horizontal, making it more difficult to see completely. By one month of age the drum is in the oblique (more vertical) position as in the adult. During the first few days after the birth, the tympanic membrane of a newborn often appears thickened and opaque. It may look injected and have a mild redness from increased vascularity, but it is not due to infection. The eardrum of a neonated is more horizontal (not oblique), making it more difficult to see completely. By one month of age the drum is in the oblique (more vertical) position as in the adult.

A 92-year-old patient has had a stroke. The right side of his face is drooping. The nurse might also suspect which of these assessment findings? a. Epistaxis b. Rhinorrhea c. Dysphagia d. Xerostomia

ANS: C Dysphagia is difficulty with swallowing and may occur with a variety of disorders, including stroke and other neurologic diseases. Rhinorrhea is a runny nose, epistaxis is a bloody nose, and xerostomia is a dry mouth. Rhinorrhea is a runny nose, epistaxis is a bloody nose, and xerostomia is a dry mouth; none of which are expected findings in a patient who had a stroke with drooping on the right side of the face. Dysphagia is difficulty with swallowing and may occur with a variety of disorders, including stroke and other neurologic diseases.

What is the primary purpose of the ciliated mucous membrane in the nose? a. To warm the inhaled air b. To filter out dust and bacteria c. To filter coarse particles from inhaled air d. To facilitate the movement of air through the nares

ANS: C The nasal hairs, or cilia, filter the coarsest matter from inhaled air, whereas the mucous blanket filters out dust and bacteria. The rich blood supply of the nasal mucosa warms the inhaled air. The rich blood supply of the nasal mucosa warms the inhaled air, not the ciliated mucous membrane. The mucous blanket, not the cilia, filters out dust and bacteria. The cilia in the nose do not facilitate the movement of air through the nares. Instead, the nasal hairs, or cilia, filter the coarsest matter from inhaled air.

The nurse notices that the mother of a 2-year-old boy brings him into the clinic quite frequently for various injuries and suspects there may be some child abuse involved. What should the nurse look for during an inspection of this child's mouth? a. Swollen, red tonsils b. Ulcerations on the hard palate c. Bruising on the buccal mucosa or gums d. Small yellow papules along the hard palate

ANS: C The nurse should notice any bruising or laceration on the buccal mucosa or gums of an infant or young child. Trauma may indicate child abuse from a forced feeding of a bottle or spoon.

The nurse is performing an otoscopic examination on an adult. Which of these actions is correct? a. Tilt the person's head forward during the examination. b. Once the speculum is in the ear, releasing the traction. c. Pulling the pinna up and back before inserting the speculum. d. Using the smallest speculum to decrease the amount of discomfort.

ANS: C The pinna is pulled up and back on an adult or older child, which helps straighten the S-shape of the canal. Traction should not be released on the ear until the examination is completed and the otoscope is removed. The nurse should tilt the patient's head slightly away from them and toward the opposite shoulder, not forward. The traction on the pinna of the ear should not be released until the examination is finished and the otoscope has been removed. The largest speculum that fits comfortably in the ear, not the smallest, should be used. The correct action is to pull the pinna up and back on an adult or older child (down and back on an infant or child under the age of 3), which helps straighten the S-shape of the canal.

What are the projections in the nasal cavity that increase the surface area are called? a. Meatus b. Septum c. Turbinates d. Kiesselbach plexus

ANS: C The projections in the nasal cavity that increases the surface area are called turbinates. The lateral walls of each nasal cavity contain three parallel bony projections: the superior, middle, and inferior turbinates. These increase the surface area, making more blood vessels and mucous membrane available to warm, humidify, and filter the inhaled air. A meatus is the passageway or canal underlying each turbinate that collects drainage. The septum is what divides the nasal cavity into two slitlike air passages. The Kiesselbach plexus is a rich vascular network in the anterior part of the septum

The nurse is assessing an 80-year-old patient. Which of these findings would be expected for this patient? a. Hypertrophy of the gums b. Increased production of saliva c. Decreased ability to identify odors d. Finer and less prominent nasal hair

ANS: C The sense of smell may be reduced because of a decrease in the number of olfactory nerve fibers with aging. Nasal hairs grow coarser and stiffer with aging. The gums may recede with aging, not hypertrophy, and saliva production decreases. The gums may recede with aging, not hypertrophy, and saliva production decreases. Nasal hairs grow coarser and stiffer with aging. Because of a decrease in the number of olfactory nerve fibers with aging, the nurse should expect a reduced sense of smell in this patient.

The nurse assesses the hearing of a 7-month-old by clapping hands. What is the expected response? a. The infant shows no obvious response to the noise. b. The infant shows a startle and acoustic blink reflex. c. The infant turns his or her head to localize the sound. d. The infant stops any movement, and appears to listen for the sound.

ANS: C With a loud sudden noise, the nurse should notice the infant turning his or her head to localize the sound and to respond to his or her own name. A startle reflex and acoustic blink reflex is expected in newborns; at age 3 to 4 months, the infant stops any movement and appears to listen. A 7-month-old infant should respond to noise. With a loud sudden noise, the nurse should notice the infant turning his or her head (not stopping any movement) to localize the sound and to respond to his or her own name.

The nurse is performing an assessment on a 21-year-old patient and notices that his nasal mucosa appears pale, gray, and swollen. What would be the most appropriate question to ask the patient? a. "Have you had any symptoms of a cold?" b. "Do you have an elevated temperature?" c. "Are you aware of having any allergies?" d. "Have you been having frequent nosebleeds?"

ANS: C With chronic allergies, the mucosa looks swollen, boggy, pale, and gray. Elevated body temperature, colds, and nosebleeds do not cause these mucosal changes. Elevated body temperature, colds, and nosebleeds do not cause the nasal mucosa to appear pale, gray, and swollen. Chronic allergies do cause the mucosa to look swollen, boggy, pale, and gray.

The nurse notices that an infant has a large, soft lump on the side of his head and that his mother is very concerned. The mother tells the nurse that she noticed the lump approximately 8 hours after her baby's birth and that it seems to be getting bigger. What is a possible explanation for this? a. Hydrocephalus b. Craniosynostosis c. Cephalhematoma d. Caput succedaneum

ANS: C A cephalhematoma is a subperiosteal hemorrhage that is the result of birth trauma. It is soft, fluctuant, and well defined over one cranial bone. It appears several hours after birth and gradually increases in size. Hydrocephalus is enlarged head due to increased cerebral spinal fluid. Craniosynostosis is a severe deformity of the head with marked asymmetry caused by premature closure of the sutures. Caput succedaneum is edematous swelling and ecchymosis of the presenting part of the head caused by birth trauma that usually causes the skull to look markedly asymmetric.

A patient, an 85-year-old woman, is complaining about the fact that the bones in her face have become more noticeable. What explanation should the nurse give her? a. Diets low in protein and high in carbohydrates may cause enhanced facial bones. b. Bones can become more noticeable if the person does not use a dermatologically approved moisturizer. c. More noticeable facial bones are probably due to a combination of factors r/t aging, such as decreased elasticity, subcutaneous fat, and moisture in her skin. d. Facial skin becomes more elastic with age. This increased elasticity causes the skin to be more taught, drawing attention to the facial bones.

ANS: C A low protein, high carbohydrate diet do not enhance facial bones; although aging adults have diminished moisture in their skin, the bones do not become more noticeable; and the elasticity of the skin decreases, not increases, with aging. The facial bones and orbits appear more prominent in the aging adult and the facial skin sags, which is attributable to decreased elasticity, decreased subcutaneous fat, and decreased moisture in the skin.

While performing the otoscopic examination of a 3-year-old boy who has been pulling on his left ear, the nurse finds that his left tympanic membrane is bright red and that the light reflex is not visible. What do these findings indicate? a. Cholesteatoma b. A fungal infection c. An acute otitis media d. A perforation of the eardrum

ANS: C Absent or distorted light reflex and a bright red color of the eardrum are indicative of acute otitis media. Cholesteatoma is an overgrowth of epidermal tissue in the middle ear or temporal bone that has a pearly white, cheesy appearance (not bright red). A fungal infection manifests as a colony of black or white dots on the eardrum or canal walls (not bright red). A perforated eardrum usually appears as a round or oval darkened area on the drum. This patient's absent light reflex and bright red color indicate acute otitis media.

During an examination of a female patient, the nurse notes lymphadenopathy and suspects an acute infection. How do acutely infected lymph nodes typically appear? a. Clumped b. Unilateral c. Firm but freely movable d. Soft and nontender

ANS: C Acutely infected lymph nodes are bilateral, enlarged, warm, tender, and firm but freely movable. Unilaterally enlarged nodes that are firm and nontender may indicate cancer.

A patient with a middle ear infection asks the nurse, "What does the middle ear do?" Which is the best response by the nurse? a. It helps maintain balance. b. It interprets sounds as they enter the ear. c. It conducts vibrations of sounds to the inner ear. d. It increases the amplitude of sound for the inner ear to function.

ANS: C Among its other functions, the middle ear conducts sound vibrations from the outer ear to the central hearing apparatus in the inner ear. The other responses are not functions of the middle ear. The inner ear, not the middle ear, helps with balance. Sound is interpreted in the cerebral cortex, not the middle ear. The middle ear reduces the amplitude of loud sounds, not increase them, to protect the inner year. The functions of the middle ear are to conduct sound vibrations from the outer ear to the central hearing apparatus in the inner ear; protect the inner ear by reducing the amplitude of loud sounds; and allow equalization of air pressure on each side of the tympanic membrane via the eustachian tubes so that the membrane does not rupture.

During an examination of a 3-year-old child, the nurse notices a bruit over the left temporal area. What should the nurse do? a. Check for the bruit again in 1 hour. b. Stop the examination, and notify the physician. c. Continue the examination because a bruit is a normal finding for this age. d. Notify the parents that a bruit has been detected in their child and requires further evaluation.

ANS: C Bruits are common in the skull in children under 4 or 5 years of age and in children with anemia. They are systolic or continuous and are heard over the temporal area. There is no need to stop the examination and notify the physician, check the bruit in an hour, or further evaluation as bruits are common in the skull in children under 4 or 5 years of age and in children with anemia. They are systolic or continuous and are heard over the temporal area

During an assessment of a 26-year-old for "a spot on my lip I think is cancer," the clinic nurse notices a group of clear vesicles with an erythematous base around them located at the lip-skin border. The patient mentions that she just returned from Hawaii. What is the most appropriate action by the nurse? a. Tell the patient she needs to see a skin specialist. b. Discuss the benefits of having a biopsy performed on any unusual lesion. c. Tell the patient that these vesicles are indicative of herpes simplex I or cold sores and that they will heal in 4 to 10 days. d. Tell the patient that these vesicles are most likely the result of a riboflavin deficiency and discuss nutrition.

ANS: C Cold sores are groups of clear vesicles with a surrounding erythematous base. These evolve into pustules or crusts and heal in 4 to 10 days. The most likely site is the lip-skin junction. Infection often recurs in the same site. Recurrent herpes infections may be precipitated by sunlight, fever, colds, or allergy

Immediately after birth, the nurse is unable to suction the nares of a crying newborn. An attempt is made to pass a catheter through both nasal cavities with no success. What should the nurse do next? a. Attempt to suction again with a bulb syringe. b. Wait a few minutes, and try again once the infant stops crying. c. Recognize that this situation requires immediate intervention. d. Contact the physician to schedule an appointment for the infant at his or her next hospital visit.

ANS: C Determining the patency of the nares in the immediate newborn period is essential because most newborns are obligate nose breathers. Nares blocked with amniotic fluid are gently suctioned with a bulb syringe. If obstruction is suspected, then a small lumen (5 to 10 Fr) catheter is passed down each naris to confirm patency. The inability to pass a catheter through the nasal cavity indicates choanal atresia, which requires immediate intervention.

A visitor from Poland who does not speak English seems to be somewhat apprehensive about the nurse examining his neck. How should the nurse proceed that would allow the patient to feel more comfortable with the nurse examining his thyroid gland? a. Behind with the nurse's hands placed firmly around his neck b. The side with the nurse's eyes averted toward the ceiling and thumbs on his neck c. The front with the nurse's thumbs placed on either side of his trachea and his head tilted forward d. The front with the nurse's thumbs placed on either side of his trachea and his head tilted backward

ANS: C Examining this patient's thyroid gland from the back may be unsettling for him. It would be best to examine his thyroid gland using the anterior approach, asking him to tip his head forward and to the right and then to the left.

A mother asks when her newborn infant's eyesight will be developed. What is the best response by the nurse? a. "Vision is not totally developed until 2 years of age." b. "Infants develop the ability to focus on an object at approximately 8 months of age." c. "By approximately 3 months of age, infants develop more coordinated eye movements and can fixate on an object." d. "Most infants have uncoordinated eye movements for the first year of life."

ANS: C Eye movements may be poorly coordinated at birth, but by 3 to 4 months of age, the infant should establish binocularity and should be able to fixate simultaneously on a single image with both eyes.

During a well-baby checkup, the nurse notices that a 1-week-old infant's face looks small compared with his cranium, which seems enlarged. On further examination, the nurse also notices dilated scalp veins and downcast or "setting sun" eyes. The nurse suspects which condition? a. Craniotabes b. Microcephaly c. Hydrocephalus d. Caput succedaneum

ANS: C Hydrocephalus occurs with the obstruction of drainage of cerebrospinal fluid that results in excessive accumulation, increasing intracranial pressure, and an enlargement of the head. The face looks small, compared with the enlarged cranium, and dilated scalp veins and downcast or "setting sun" eyes are noted. Craniotabes is a softening of the skull's outer layer. Microcephaly is an abnormally small head. A caput succedaneum is edematous swelling and ecchymosis of the presenting part of the head caused by birth trauma. The signs and symptoms of the infant in this question are those of hydrocephalus.

During an interview, the patient states he has the sensation that "everything around him is spinning." What part of the ear should the nurse recognize is responsible for this sensation? a. Cochlea b. CN VIII c. Labyrinth d. Organ of Corti

ANS: C If the labyrinth of the ear becomes inflamed, it feeds the wrong information to the brain, creating a staggering gait and a strong, spinning, whirling sensation called vertigo. The cochlea, which contains the central hearing apparatus, and cranial nerve VIII, the vestibulocochlear nerve, which conducts nerve impulses from the organ of Corti to the brain, are all involved with hearing. The spinning sensation that this patient is experiencing is from the labyrinth of the ear

During an examination of the eye, the nurse would expect what normal finding when assessing the lacrimal apparatus? a. Presence of tears along the inner canthus b. Blocked nasolacrimal duct in a newborn infant c. Absence of drainage from the puncta when pressing against the inner orbital rim d. Slight swelling over the upper lid and along the bony orbit if the individual has a cold

ANS: C No swelling, redness, or drainage from the puncta should be observed when it is pressed. Regurgitation of fluid from the puncta, when pressed, indicates duct blockage. The lacrimal glands are not functional at birth.

The nurse notices the presence of periorbital edema when performing an eye assessment on a 70-year-old patient. What should the nurse do next? a. Check for the presence of exophthalmos. b. Suspect that the patient has hyperthyroidism. c. Ask the patient if he or she has a history of heart failure. d. Assess for blepharitis, which is often associated with periorbital edema.

ANS: C Periorbital edema occurs with local infections, crying, and systemic conditions such as heart failure, renal failure, allergy, and hypothyroidism. Periorbital edema is not associated with blepharitis. Exophthalmos is associated with hyperthyroidism or thyrotoxicosis and hyperthyroidism is not associated with periorbital edema (although hypothyroidism is). Periorbital edema is not associated with blepharitis, either. Thus, the nurse should ask about these conditions.

What is the nurse assessing for when he or she directs a light across the iris of a patient's eye from the temporal side? a. Drainage from dacryocystitis b. Presence of conjunctivitis over the iris c. Presence of shadows, which may indicate glaucoma d. Scattered light reflex, which may be indicative of cataracts

ANS: C The presence of shadows in the anterior chamber may be a sign of acute angle-closure glaucoma. The normal iris is flat and creates no shadows. This technique (directing a light across the iris of a patient's eye from the temporal side) is not the technique to assess for dacryocystitis, conjunctivitis, or cataracts.

A 70-year-old patient tells the nurse that he has noticed that he is having trouble hearing, especially in large groups. He says "I can't always tell where the sound is coming from" and that the words often sound "mixed up." What might the nurse suspect as the cause for this change? a. Atrophy of the apocrine glands b. Cilia becoming coarse and stiff c. Nerve degeneration in the inner ear d. Scar tissue in the tympanic membrane

ANS: C Presbycusis is a type of hearing loss that occurs in 60% of those older than 65 years of age, even in those living in a quiet environment. This sensorineural loss is gradual and caused by nerve degeneration in the inner ear. Words sound garbled, and the ability to localize sound is also impaired. This communication dysfunction is accentuated when background noise is present. Atrophy of the apocrine glands causes cerumen to be more dry and cilia becoming coarse and stiff may cause cerumen to accumulate and oxidize and reduce hearing but they do cause this patient's symptoms of not being able to locate the source of sounds or sounds being mixed up. Scarring of the tympanic eardrum are sequelae of repeated ear infections but do not necessarily affect hearing.

A 60-year-old man is at the clinic for an eye examination. The nurse suspects that he has ptosis of one eye. How should the nurse check for this? a. Perform the confrontation test. b. Assess the individual's near vision. c. Observe the distance between the palpebral fissures. d. Perform the corneal light test, and look for symmetry of the light reflex

ANS: C Ptosis is a drooping of the upper eyelid that would be apparent by observing the distance between the upper and lower eyelids. The confrontation test measures peripheral vision, not ptosis. Measuring near vision or the corneal light test does not check for ptosis.

A patient has been identified as having a sensorineural hearing loss. What would be important for the nurse to do during the assessment of this patient? a. Speak loudly so the patient can hear the questions. b. Assess for middle ear infection as a possible cause. c. Ask the patient what medications he is currently taking. d. Look for the source of the obstruction in the external ear.

ANS: C Sensorineural hearing loss may be caused by presbycusis, which is a gradual nerve degeneration that occurs with aging and by ototoxic drugs, which affect the hair cells in the cochlea. So the nurse should ask the patients about the medications they have been taking. A simple increase in amplitude may not enable the person to understand spoken words. The middle ear and obstruction of the external ear are not associated with sensorineural hearing loss so the nurse should not assess for a middle ear infection or external ear obstruction.

The nurse is preparing to assess the visual acuity of a 16-year-old patient. How should the nurse proceed? a. Perform the confrontation test. b. Ask the patient to read the print on a handheld Jaeger card. c. Use the Snellen chart positioned 20 feet away from the patient. d. Determine the patient's ability to read newsprint at a distance of 12 to 14 inches

ANS: C The Snellen alphabet chart is the most commonly used and most accurate measure of visual acuity. The confrontation test is a gross measure of peripheral vision. The Jaeger card or newspaper tests are used to test near vision.

The nurse is examining a patient's ears and notices cerumen in the external canal. Which of these statements about cerumen is correct? a. Wet, honey-colored cerumen is a sign of infection. b. The presence of cerumen is indicative of poor hygiene. c. The purpose of cerumen is to protect and lubricate the ear. d. Cerumen is necessary for transmitting sound through the auditory canal.

ANS: C The ear is lined with glands that secrete cerumen. Cerumen is genetically determined, with two distinct types. Wet, honey-brown occurs in Caucasians and African Americans, and a dry, flaky white is found in East Asians and American Indians. Cerumen is supposed to be present-to lubricate, waterproof, and clean the external auditory canal. It also is antibacterial, and traps foreign bodies. Wet, honey-colored cerumen is not a sign of infection. Cerumen is not a sign of poor hygiene. It is supposed to be present-to lubricate, waterproof, and clean the external auditory canal. It also is antibacterial, and traps foreign bodies. It is not necessary for transmitting sound through the auditory canal and too much cerumen can impair hearing.

A patient comes to the clinic complaining of neck and shoulder pain and is unable to turn her head. Which nerve does the nurse suspect is damaged and how should the nurse proceed with the examination? a. XII; assess for a positive Romberg sign. b. XI; palpate the anterior and posterior triangles. c. XI; have patient shrug their shoulders against resistance. d. XII; percuss the sternomastoid and submandibular neck muscles

ANS: C The major neck muscles are the sternomastoid and the trapezius. They are innervated by CN XI, the spinal accessory. The innervated muscles assist with head rotation and head flexion, movement of the shoulders, and extension and turning of the head. To assess the function of cranial nerve XI the nurse should have the patient shrug their shoulders against resistance. Cranial nerve XII is the hypoglossal nerve which innervates the muscles of the tongue involved with speech and swallowing and is not involved in head movement. Identifying the anterior and posterior triangles are helpful guidelines when describing findings in the neck but palpating them does not assess any cranial nerves.

The nurse is testing a patient's visual accommodation. How is accommodation assessed? a. Pupillary dilation when looking at a distant object b. Involuntary blinking in the presence of bright light c. Pupillary constriction when looking at a near object d. Changes in peripheral vision in response to bright light

ANS: C The muscle fibers of the iris contract the pupil in bright light and accommodate for near vision, which also results in pupil constriction. The other responses are not correct.

The nurse is reviewing the function of the cranial nerves (CNs). Which CN is responsible for conducting nerve impulses to the brain from the organ of Corti? a. I b. III c. VIII d. XI

ANS: C The nerve impulses are conducted by the auditory portion of CN VIII to the brain. Cranial nerve I, the olfactory nerve, is responsible for the sense of smell. Cranial nerve III, the oculomotor, innervates the superior, inferior, and medial rectus and the inferior oblique muscles of the eye. Cranial nerve XI, the accessory nerve, controls the muscles of the neck. The nerve that conducts nerve impulses from the organ of Corti to the brain is CN VIII, the vestibulocochlear nerve.

In using the ophthalmoscope to assess a patient's eyes, the nurse notices a red glow in the patient's pupils. Based on this finding, what should the nurse do? a. Suspect that an opacity is present in the lens or cornea. b. Check the light source of the ophthalmoscope to verify that it is functioning. c. Consider the red glow a normal reflection of the ophthalmoscope light off the inner retina. d. Continue with the ophthalmoscopic examination, and refer the patient for further evaluation.

ANS: C The red glow filling the person's pupil is the red reflex and is a normal finding caused by the reflection of the ophthalmoscope light off the inner retina, so there is no need to check the light source of the ophthalmoscope or referral of the patient, and the interruption or absence, not the presence, of the red reflex would indicate an opacity. The other responses are not correct.

The nurse needs to palpate the temporomandibular joint for crepitation. Where is this joint located? a. Just below the hyoid bone and posterior to the tragus b. Just below the vagus nerve and posterior to the mandible c. Just below the temporal artery and anterior to the tragus d. Just below the temporal artery and anterior to the mandible

ANS: C The temporomandibular joint is just below the temporal artery and anterior to the tragus.

During an examination of a patient in her third trimester of pregnancy, the nurse notices that the patient's thyroid gland is slightly enlarged. No enlargement had been previously noticed. What does the nurse suspect? a. An iodine deficiency b. Early signs of goiter c. A normal enlargement of the thyroid gland during pregnancy d. Possible thyroid cancer and the need for further evaluation

ANS: C The thyroid gland enlarges slightly during pregnancy because of hyperplasia of the tissue and increased vascularity.

The physician reports that a patient with a neck tumor has a tracheal shift. The nurse should understand that what is occurring to the patient's trachea? a. Pushed downward b. Pulled to the affected side c. Pushed to the unaffected side d. Pulled downward in a rhythmic pattern

ANS: C The trachea is pushed to the unaffected side with an aortic aneurysm, a tumor, unilateral thyroid lobe enlargement, or a pneumothorax. The trachea is pulled to the affected side with large atelectasis, pleural adhesions, or fibrosis. Tracheal tug is a rhythmic downward pull that is synchronous with systole and occurs with aortic arch aneurysm. The trachea is pulled to the affected side with large atelectasis, pleural adhesions, or fibrosis. Tracheal tug is a rhythmic downward pull that is synchronous with systole and occurs with aortic arch aneurysm. The trachea is pushed to the unaffected side with an aortic aneurysm, a tumor, unilateral thyroid lobe enlargement, or a pneumothorax

While performing a well-child assessment on a 5 year old, the nurse notes the presence of palpable, bilateral, cervical, and inguinal lymph nodes. They are approximately 0.5 cm in size, round, mobile, and nontender. What do these findings lead the nurse to conclude? a. The child has chronic allergies. b. The child likely has an infection. c. These are normal findings for a well child of this age. d. These findings indicate a need for additional evaluation.

ANS: C These are not signs of chronic allergies or an infection and do not require additional evaluation. Palpable lymph nodes are normal in children until puberty when the lymphoid tissue begins to atrophy. Lymph nodes may be up to 1 cm in size in the cervical and inguinal areas but are discrete, movable, and nontender.

During an examination, the patient states he is hearing a buzzing sound and says that it is "driving me crazy!" What does this indicate? a. Vertigo b. Pruritus c. Tinnitus d. Cholesteatoma

ANS: C Tinnitus is a sound that comes from within a person; it can be a ringing, crackling, or buzzing sound. It accompanies some hearing or ear disorders. Vertigo is a strong spinning, whirling sensation; pruritus is itching; and cholesteatoma is an overgrowth of epidermal tissue in the middle ear or temporal bone that has a pearly white, cheesy appearance. The buzzing sound this patient is hearing is tinnitus.

When assessing the pupillary light reflex, the nurse should use which technique? a. Shine a penlight from directly in front of the patient, and inspect for pupillary constriction. b. Ask the patient to follow the penlight in eight directions, and observe for bilateral pupil constriction. c. Shine a light across the pupil from the side, and observe for direct and consensual pupillary constriction. d. Ask the patient to focus on a distant object. Then ask the patient to follow the penlight to approximately 7 cm from the nose.

ANS: C To test the pupillary light reflex, the nurse should advance a light in from the side and note the direct and consensual pupillary constriction.

In a patient who has anisocoria, what would the nurse expect to observe? a. Dilated pupils b. Excessive tearing c. Pupils of unequal size d. Uneven curvature of the lens

ANS: C Unequal pupil size is termed anisocoria. It normally exists in 5% of the population but may also be indicative of central nervous system disease. Dilated pupils, excessive tearing, and uneven curvature of the lens are not associated with anisocoria. Anisocoria is the term for unequal pupil size. It exists in about 5% of the population but may also be indicative of central nervous system disease

During an examination, the nurse notices that the patient stumbles a little while walking, and, when she sits down, she holds on to the sides of the chair. The patient states, "It feels like the room is spinning!" What do this signs and symptoms indicate? a. Tinnitus b. Dizziness c. Objective vertigo d. Subjective vertigo

ANS: C With objective vertigo, the patient feels like the room spins; with subjective vertigo, the person feels like he or she is spinning. Tinnitus is a sound that comes from within a person; it can be a ringing, crackling, or buzzing sound. It accompanies some hearing or ear disorders. Dizziness is not the same as true vertigo; the person who is dizzy may feel unsteady and light-headed. With subjective vertigo, the person feels like he or she is spinning. The symptom this patient has, that the room is spinning, is objective vertigo.

A 31-year-old patient tells the nurse that he has noticed pain in his left ear when people speak loudly to him. What does this finding indicate? a. A cerumen impaction b. Normal for people of his age c. Possible middle ear infection d. A characteristic of recruitment

ANS: D Recruitment is significant hearing loss occurring when speech is at low intensity, but sound actually becomes painful when the speaker repeats at a louder volume. The other responses are not correct. A cerumen impaction and ear infection do not cause these symptoms and these are not normal findings. Instead, this patient's symptoms are a characteristic of recruitment.

During an otoscopic examination, the nurse notices an area of black and white dots on the tympanic membrane and the ear canal wall. What does this finding suggest? a. Malignancy b. Viral infection c. Blood in the middle ear d. Yeast or fungal infection

ANS: D A colony of black or white dots on the drum or canal wall suggests a yeast or fungal infection (otomycosis). A colony of black and white dots on the eardrum is not a manifestation of malignancy, a viral infection, of blood in the middle year. Blood in the middle year would cause a blue or dark red appearance of the eardrum.

A pregnant woman states that she is concerned about her gums because she has noticed they are swollen and have started to bleed. What would be an appropriate response by the nurse? a. "Your condition is probably due to a vitamin C deficiency." b. "I'm not sure what causes swollen and bleeding gums, but let me know if it's not better in a few weeks." c. "You need to make an appointment with your dentist as soon as possible to have this checked." d. "Swollen and bleeding gums can be caused by a change in hormonal balance during pregnancy."

ANS: D Although with gingivitis (which can be caused by a vitamin C deficiency) gum margins are red and swollen and easily bleed, a changing hormonal balance during puberty or pregnancy may also cause these symptoms. Since this patient is pregnant, a change in hormonal balance is likely the cause.

A 32-year-old woman is at the clinic for "little white bumps in my mouth." During the assessment, the nurse notes that she has a 0.5-cm white, nontender papule under her tongue and one on the mucosa of her right cheek. What would the nurse tell the patient? a. "These spots indicate an infection such as strep throat." b. "These bumps could be indicative of a serious lesion, so I will refer you to a specialist." c. "This condition is called leukoplakia and can be caused by chronic irritation such as with smoking." d. "These bumps are Fordyce granules, which are sebaceous cysts and are not a serious condition."

ANS: D Fordyce granules are small, isolated white or yellow papules on the mucosa of the cheek, tongue, and lips. These little sebaceous cysts are painless and are not significant. Chalky white, thick, raised patches would indicate leukoplakia. In strep throat, the examiner would see tonsils that are bright red, swollen, and may have exudates or white spots. In strep throat, the examiner would see tonsils that are bright red, swollen, and may have exudates or white spots and leukoplakia would appear as chalky white, thick, raised patches. These findings are not indicative of a serious lesion but are fordyce granules. Fordyce granules are small, isolated white or yellow papules on the mucosa of the cheek, tongue, and lips. These little sebaceous cysts are painless and are not significant.

How should the nurse perform an examination of a 2-year-old child with a suspected ear infection? a. Pull the ear up and back before inserting the speculum. b. Omit the otoscopic examination if the child has a fever. c. Ask the mother to leave the room while examining the child. d. Perform the otoscopic examination at the end of the assessment.

ANS: D In addition to its place in the complete examination, eardrum assessment is mandatory for any infant or child requiring care for an illness or fever. For the infant or young child, the timing of the otoscopic examination is best toward the end of the complete examination because many young children protest vigorously during this procedure and it is difficult to re-establish cooperation afterward. When performing an ear examination on a 2-year-old child, with or without a suspected ear infection, the pinna should be pulled down (not up) and back. In addition to its place in the complete examination, eardrum assessment is mandatory for any infant or child requiring care for an illness or fever, so should not be omitted. Rather than asking the parent to leave the room, the nurse should enlist the parent's help in holding the child to protect the eardrum from injury

During an admission assessment, the nurse notices that a male patient has an enlarged and rather thick skull. The nurse suspects acromegaly. What additional finding would the nurse assess for to confirm this suspicion? a. Exophthalmos b. Bowed long bones c. Acorn-shaped cranium d. Coarse facial features

ANS: D Acromegaly is the excessive secretion of growth hormone that creates an enlarged skull and thickened cranial bones. Patients will have elongated heads, massive faces, prominent noses and lower jaws, heavy eyebrow ridges, and coarse facial features. Exophthalmos is associated with hyperthyroidism. Bowed long bones and an acorn-shaped cranium result from Paget disease

The nurse is obtaining a health history on a 3-month-old infant. During the interview, the mother states, "I think she is getting her first tooth because she has started drooling a lot." What is the best response by the nurse? a. "You're right, drooling is usually a sign of the first tooth." b. "It would be unusual for a 3-month-old to be getting her first tooth." c. "This could be the sign of a problem with the salivary glands." d. "She is just starting to salivate and hasn't learned to swallow the saliva."

ANS: D In the infant, salivation starts at 3 months. The baby will drool for a few months before learning to swallow the saliva. This drooling does not herald the eruption of the first tooth, although many parents think it does. Although many parents think the start of drooling signals the eruption of the first tooth, it does not. Although teeth usually erupt between 6 and 24 months, the nurse should not just say it would be unusual for a 3-month-old to be getting her first tooth as that does not address the issue of the drooling. It is also not a sign of a problem.

A woman who is in the second trimester of pregnancy mentions that she has had "more nosebleeds than ever" since she became pregnant. What is the likely reason for this? a. Inappropriate use of nasal sprays b. A problem with the patient's coagulation system c. Increased susceptibility to colds and nasal irritation d. Increased vascularity in the upper respiratory tract as a result of the pregnancy

ANS: D Nasal stuffiness and epistaxis may occur during pregnancy as a result of increased vascularity in the upper respiratory tract. Inappropriate use of nasal sprays often causes rebound congestion or swelling, but not usually nosebleeds. Nasal stuffiness and epistaxis may occur during pregnancy as a result of increased vascularity in the upper respiratory tract so this patient's nose bleeds are more likely to be due to the increased vascularity in the upper respiratory tract than to a problem with the coagulation system or an increased susceptibility to colds and nasal irritation.

The nurse is reviewing the development of the newborn infant. Regarding the sinuses, which statement is true in relation to a newborn infant? a. Sphenoid sinuses are full size at birth. b. Maxillary sinuses reach full size after puberty. c. Frontal sinuses are fairly well developed at birth. d. Maxillary and ethmoid sinuses are the only sinuses present at birth.

ANS: D Only the maxillary and ethmoid sinuses are present at birth. The sphenoid sinuses are minute at birth and develop after puberty. The frontal sinuses are absent at birth, are fairly well developed at age 7 to 8 years, and reach full size after puberty. The sphenoid sinuses are minute at birth and develop after puberty. The frontal sinuses are absent at birth, are fairly well developed at age 7 to 8 years, and reach full size after puberty. Only the maxillary and ethmoid sinuses are present at birth but the maxillary sinus does not reach full size until all permanent teeth have erupted (not after puberty).

While obtaining a health history from the mother of a 1-year-old child, the nurse notices that the baby has had a bottle in his mouth the entire time. The mother states, "It makes a great pacifier." What is the best response by the nurse? a. "You're right. Bottles make very good pacifiers." b. "Using a bottle as a pacifier is better for the teeth than thumb-sucking." c. "It's okay to use a bottle as long as it contains milk and not juice." d. "Prolonged use of a bottle can increase the risk for tooth decay and ear infections."

ANS: D Prolonged bottle use during the day or when going to sleep places the infant at risk for tooth decay and middle ear infections.

An assessment of a 23-year-old patient reveals the following: an auricle that is tender and reddish-blue in color with small vesicles. The nurse would need to know additional information that includes which of these? a. Any change in ability to hear b. Any recent drainage from the ear c. Recent history of trauma to the ear d. Any prolonged exposure to extreme cold

ANS: D Reddish-blue discoloration and swelling of the auricle are manifestations of frostbite so the nurse should ask about any prolonged exposure to extreme cold rather than changes in ability to hear or drainage or recent trauma to the ear. Frostbite causes reddish-blue discoloration and swelling of the auricle after exposure to extreme cold. Vesicles or bullae may develop, and the person feels pain and tenderness.

When examining the nares of a 45-year-old patient who is experiencing rhinorrhea, itching of the nose and eyes, and sneezing, the nurse notices the following: pale turbinates, swelling of the turbinates, and clear rhinorrhea. Which of these conditions is most likely the cause? a. Nasal polyps b. Acute rhinitis c. Acute sinusitis d. Allergic rhinitis

ANS: D Rhinorrhea, itching of the nose and eyes, and sneezing are manifestations of allergic rhinitis. On physical examination, serous edema is noted, and the turbinates usually appear pale with a smooth, glistening surface. Nasal polyps appear as smooth, pale gray nodules which are overgrowths of mucosa most commonly caused by chronic allergic rhinitis and often cause absence of sense of smell and a sensation of a "valve that moves" in the nose when breathing. Acute rhinitis initially presents with clear, watery discharge (rhinorrhea) which later become purulent, with sneezing nasal itching, stimulation of cough reflex, and inflamed mucosa with dark red and swollen turbinates which cause nasal obstruction. With sinusitis, there is usually mucopurulent drainage, nasal obstruction, facial pain or pressure, and may have fever, chills, and malaise. This patient's symptoms of rhinorrhea, itching of the nose and eyes, and sneezing are manifestations of allergic rhinitis. On physical examination, serous edema is noted, and the turbinates usually appear pale with a smooth, glistening surface.

A mother brings her 4-month-old infant to the clinic with concerns regarding a small pad in the middle of the upper lip that has been there since 1 month of age. The infant has no health problems. On physical examination, the nurse notices a 0.5-cm, fleshy, elevated area in the middle of the upper lip. No evidence of inflammation or drainage is observed. What would the nurse tell this mother? a. "This area of irritation is caused from teething and is nothing to worry about." b. "This finding is abnormal and should be evaluated by another health care provider." c. "This area of irritation is the result of chronic drooling and should resolve within the next month or two." d. "This elevated area is a sucking tubercle caused from the friction of breastfeeding or bottle-feeding and is normal."

ANS: D A normal finding in infants is the sucking tubercle, a small pad in the middle of the upper lip from the friction of breastfeeding or bottle-feeding. This condition is not caused by irritation, teething, or excessive drooling, and evaluation by another health care provider is not warranted.

A patient comes into the clinic complaining of facial pain, fever, and malaise. On examination, the nurse notes swollen turbinates and purulent discharge from the nose. The patient also complains of a dull, throbbing pain in his cheeks and teeth on the right side and pain when the nurse palpates the areas. What do these findings indicate? a. Nasal polyps b. Frontal sinusitis c. Posterior epistaxis d. Maxillary sinusitis

ANS: D Signs of maxillary sinusitis include facial pain after upper respiratory infection, red swollen nasal mucosa, swollen turbinates, and purulent discharge. The person also has fever, chills, and malaise. With maxillary sinusitis, dull throbbing pain occurs in the cheeks and teeth on the same side, and pain with palpation is present. With frontal sinusitis, pain is above the supraorbital ridge. Nasal polyps appear as smooth, pale gray nodules which are overgrowths of mucosa most commonly caused by chronic allergic rhinitis and often cause absence of sense of smell and a sensation of a "valve that moves" in the nose when breathing. Epistaxis is a nosebleed and the most common site of bleeding is the Kiesselbach plexus in the anterior septum. With frontal sinusitis, pain is above the supraorbital ridge. This patient's signs and symptoms are indicative of maxillary sinusitis. Signs of maxillary sinusitis include facial pain after upper respiratory infection, red swollen nasal mucosa, swollen turbinates, and purulent discharge. The person also has fever, chills, and malaise. With maxillary sinusitis, dull throbbing pain occurs in the cheeks and teeth on the same side, and pain with palpation is present.

What is the tissue that connects the tongue to the floor of the mouth called? a. Uvula b. Palate c. Papillae d. Frenulum

ANS: D The frenulum is a midline fold of tissue that connects the tongue to the floor of the mouth. The uvula is the free projection hanging down from the middle of the soft palate. The palate is the arching roof of the mouth. Papillae are the rough, bumpy elevations on the tongue's dorsal surface. The uvula is the free projection hanging down from the middle of the soft palate. The palate is the arching roof of the mouth. Papillae are the rough, bumpy elevations on the tongue's dorsal surface. The frenulum is a midline fold of tissue that connects the tongue to the floor of the mouth

The mother of a 2-year-old is concerned because her son has had three ear infections in the past year. What would be an appropriate response by the nurse? a. "It is unusual for a small child to have frequent ear infections unless something else is wrong." b. "We need to check the immune system of your son to determine why he is having so many ear infections." c. "Ear infections are not uncommon in infants and toddlers because they tend to have more cerumen in the external ear." d. "Your son's eustachian tube is shorter and wider than yours because of his age, which allows for infections to develop more easily."

ANS: D The infant's eustachian tube is relatively shorter and wider than the adult's eustachian tube, and its position is more horizontal; consequently, pathogens from the nasopharynx can more easily migrate through to the middle ear. The other responses are not appropriate. It is not unusual for a small child to have frequent ear infections, thus, it is not necessary to check the immune system. The reason that ear infections in infants and toddlers is not uncommon is not due to more cerumen but because the infant's eustachian tubes are relatively shorter and wider than the adult's eustachian tube, and its position is more horizontal; consequently, pathogens from the nasopharynx can more easily migrate through to the middle ear.

In an individual with otitis externa, which of these signs would the nurse expect to find on assessment? a. Rhinorrhea b. Periorbital edema c. Pain over the maxillary sinuses d. Enlarged superficial cervical nodes

ANS: D The lymphatic drainage of the external ear flows to the parotid, mastoid, and superficial cervical nodes. The signs are severe swelling of the canal, inflammation, and tenderness. Rhinorrhea, periorbital edema, and pain over the maxillary sinuses do not occur with otitis externa.

During a checkup, a 22-year-old woman tells the nurse that she uses an over-the-counter nasal spray because of her allergies. She also states that it does not work as well as it used to when she first started using it. Which is the best response by the nurse? a. "You should never use over-the-counter nasal sprays because of the risk for addiction." b. "You should try switching to another brand of medication to prevent this problem." c. "Continuing to use this spray is important to keep your allergies under control." d. "Frequent use of these nasal medications irritates the lining of the nose and may cause rebound swelling."

ANS: D The misuse of over-the-counter nasal medications irritates the mucosa and causes the blood vessels to become swollen, rebound swelling, which is a common problem.

The nurse is assessing a patient with a history of intravenous drug abuse. In assessing his mouth, the nurse notices a dark red confluent macule on the hard palate. This could be an early sign of what disease or disorder? a. Measles b. Leukemia c. A carcinoma d. Acquired immunodeficiency syndrome (AIDS)

ANS: D This dark red confluent macule on the hard palate is an oral Kaposi's sarcoma. An oral Kaposi's sarcoma is a bruiselike, dark red or violet, confluent macule that usually occurs on the hard palate but may also appear on the soft palate or gingival margin. Oral lesions such as a Kaposi's sarcoma are among the earliest lesions to develop with AIDS.

The nurse is taking the history of a patient who may have a perforated eardrum. What would be an important question in this situation? a. "Do you ever notice ringing or crackling in your ears?" b. "When was the last time you had your hearing checked?" c. "Have you ever been told that you have any type of hearing loss?" d. "Is there any relationship between the ear pain and the discharge you mentioned?"

ANS: D Typically with perforation, ear pain occurs first and resolves after a popping sensation, then drainage occurs.

The nurse is performing an assessment. Which of these findings would cause the greatest concern? a. A painful vesicle inside the cheek for 2 days b. The presence of moist, nontender Stensen's ducts c. Stippled gingival margins that snugly adhere to the teeth d. An ulceration on the side of the tongue with rolled edges

ANS: D Ulceration on the side or base of the tongue or under the tongue raises the suspicion of cancer and must be investigated. The risk for early metastasis is present because of rich lymphatic drainage. The vesicle may be an aphthous ulcer, which is painful but not dangerous. The other responses are normal findings. The presence of moist, nontender Stensen's ducts and stippled gingival margins that snugly adhere to the teeth are normal findings. Although a painful vesicle inside the cheek for 2 days is not that uncommon or concerning, but an ulceration on the side, base, or under the tongue raises the suspicion of cancer and must be investigated. The risk for early metastasis is present because of rich lymphatic drainage. The vesicle may be an aphthous ulcer, which is painful but not dangerous.

The nurse is assessing color vision of a male child. Which statement is correct? a. Color vision should be checked annually until the age of 18 years. b. Color vision screening should begin at the child's 2-year checkup. c. The nurse should ask the child to identify the color of his or her clothing. d. Testing for color vision should be done once between the ages of 4 and 8 years.

ANS: D Boys should be tested only once for color vision between the ages of 4 and 8 years. Color vision is not tested in girls because it is rare in girls. Testing is performed with the Ishihara test, which is a series of polychromatic cards.

A 52-year-old patient describes the presence of occasional floaters or spots moving in front of his eyes. How should the nurse proceed? a. Examine the retina to determine the number of floaters. b. Presume the patient has glaucoma and refer him for further testing. c. Consider these to be abnormal findings, and refer him to an ophthalmologist. d. Understand that floaters are usually insignificant and are caused by condensed vitreous fibers.

ANS: D Floaters are a common sensation with myopia or after middle age and are attributable to condensed vitreous fibers. Floaters or spots are not usually significant, but the acute onset of floaters may occur with retinal detachment. The floaters or spots would not be visible for the nurse to see or count. A decrease in peripheral vision is a symptom of glaucoma, not floaters.

A patient is unable to read even the largest letters on the Snellen chart. The nurse should take which action next? a. Refer the patient to an ophthalmologist or optometrist for further evaluation. b. Assess whether the patient can count the nurse's fingers when they are placed in front of his or her eyes. c. Ask the patient to put on his or her reading glasses and attempt to read the Snellen chart again. d. Shorten the distance between the patient and the chart until the letters are seen, and record that distance.

ANS: D If the person is unable to see even the largest letters when standing 20 feet from the chart, then the nurse should shorten the distance to the chart until the letters are seen, and record that distance (e.g., "10/200"). If visual acuity is even lower, then the nurse should assess whether the person can count fingers when they are spread in front of the eyes or can distinguish light perception from a penlight. If vision is poorer than 20/30, then a referral to an ophthalmologist or optometrist is necessary, but the nurse must first assess the visual acuity. The nurse should assess whether the person can count fingers when they are spread in front of the eyes or can distinguish light perception from a penlight only if unable to see the letters on the Snellen chart when the distance is shortened. Applying reading glasses will not help with reading the Snellen chart as that is assessing far vision, not near vision

The nurse is reviewing causes of increased intraocular pressure. Which of these factors determines intraocular pressure? a. Thickness or bulging of the lens b. Posterior chamber as it accommodates increased fluid c. Contraction of the ciliary body in response to the aqueous within the eye d. Amount of aqueous produced and resistance to its outflow at the angle of the anterior chamber

ANS: D Intraocular pressure is determined by a balance between the amount of aqueous produced and the resistance to its outflow at the angle of the anterior chamber. The other responses are incorrect.

A mother brings her 2-month-old daughter in for an examination and says, "My daughter rolled over against the wall, and now I have noticed that she has this spot that is soft on the top of her head. Is something terribly wrong?" How should the nurse respond? a. "Perhaps that could be a result of your dietary intake during pregnancy." b. "Your baby may have craniosynostosis, a disease of the sutures of the skull." c. "That 'soft spot' may be an indication of cretinism or congenital hypothyroidism." d. "That 'soft spot' is normal, and actually allows for growth of the brain during the first year of your baby's life."

ANS: D Membrane-covered "soft spots" allow for growth of the brain during the first year of life. They gradually ossify; the triangular-shaped posterior fontanel is closed by 1 to 2 months, and the diamond-shaped anterior fontanel closes between 9 months and 2 years.

A patient complains that while studying for an examination he began to notice a severe headache in the frontotemporal area of his head that is throbbing and is somewhat relieved when he lies down. He tells the nurse that his mother also had these headaches. What should the nurse suspect? a. Hypertension b. Cluster headaches c. Tension headaches d. Migraine headaches

ANS: D Migraine headaches tend to be supraorbital, retro-orbital, or frontotemporal with a throbbing quality. They are severe in quality and are relieved by lying down. Migraines are associated with a family history of migraine headaches. Although hypertension may cause headaches, the blood pressure needs to be severely elevated and would likely not be relieved with lying down. Cluster headaches produce pain around the eye, temple, forehead, and cheek and are unilateral and always on the same side of the head. They are excruciating and occur once or twice per day and last 1 2 to 2 hours each. Tension headaches are occipital, frontal, or with bandlike tightness

The nurse is explaining to a student nurse the four areas in the body where lymph nodes are accessible. Which areas should the nurse include in her explanation to the student? a. Head, breasts, groin, and abdomen b. Arms, breasts, inguinal area, and legs c. Head and neck, arms, breasts, and axillae d. Head and neck, arms, inguinal area, and axillae

ANS: D Nodes are located throughout the body, but they are accessible to examination only in four areas: head and neck, arms, inguinal region, and axillae.

An ophthalmic examination reveals papilledema. What does this finding indicate? a. Retinal detachment b. Diabetic retinopathy c. Acute-angle glaucoma d. Increased intracranial pressure

ANS: D Papilledema, or choked disk, is a serious sign of increased intracranial pressure, which is caused by a space-occupying mass such as a brain tumor or hematoma. This pressure causes venous stasis in the globe, showing redness, congestion, and elevation of the optic disc, blurred margins, hemorrhages, and absent venous pulsations. Papilledema is not associated with the conditions in the other responses. Papilledema is not associated with retinal detachment, diabetic retinopathy, or acute-angle glaucoma.

When examining a patient's eyes, what should the nurse be aware that stimulation of the sympathetic branch of the autonomic nervous system causes? a. Pupillary constriction b. Adjusts the eye for near vision c. Causes contraction of the ciliary body d. Elevates the eyelid and dilates the pupil

ANS: D Stimulation of the sympathetic branch of the autonomic nervous system dilates the pupil and elevates the eyelid. Parasympathetic nervous system stimulation causes the pupil to constrict. The muscle fibers of the iris contract the pupil in bright light to accommodate for near vision. The ciliary body controls the thickness of the lens. Parasympathetic nervous system stimulation, not sympathetic nervous system, causes the pupil to constrict. The muscle fibers of the iris contract the pupil in bright light to accommodate for near vision, not sympathetic nervous system stimulation.

The nurse is performing an eye-screening clinic at a daycare center. When examining a 2-year-old child, the nurse suspects that the child has a "lazy eye". What should the nurse do next? a. Examine the external structures of the eye. b. Assess visual acuity with the Snellen eye chart. c. Assess the child's visual fields with the confrontation test. d. Test for strabismus by performing the corneal light reflex test.

ANS: D Testing for strabismus is done by performing the corneal light reflex test and the cover test. The Snellen eye chart and confrontation test are not used to test for strabismus. Examining the external structures of the eye, assessing visual acuity with the Snellen eye chart, and assessing for confrontation are not used to test for strabismus.

The nurse is assessing a patient's eyes for the accommodation response and would expect to see which normal finding? a. Dilation of the pupils b. Consensual light reflex c. Disconjugate movement of the eyes d. Convergence of the axes of the eyes

ANS: D The accommodation reaction includes pupillary constriction and convergence of the axes of the eyes. The other responses are not correct. Dilation of the pupil occurs with dimming the lights and having the person look in the distance, not when assessing for accommodation. Consensual light reflex is simultaneous constriction of the pupil opposite the pupil that light is being shined on. Conjugate, not disconjugate, movement of the eye (the axes of both eyes remains parallel while moving) is a normal finding.

A 19-year-old college student is brought to the emergency department with a severe headache he describes as, "Like nothing I've ever had before." His temperature is 40° C, and he has a stiff neck. The nurse looks for other signs and symptoms of which problem? a. Head injury b. Cluster headache c. Migraine headache d. Meningeal inflammation

ANS: D The acute onset of neck stiffness and pain along with headache and fever occurs with meningeal inflammation. A severe headache in an adult or child who has never had it before is a red flag. Head injury and cluster or migraine headaches are not associated with a fever or stiff neck.

The nurse notices that a patient's submental lymph nodes are enlarged. In an effort to identify the cause of the node enlargement, what should the nurse assess? a. Infraclavicular area b. Supraclavicular area c. Area distal to the enlarged node d. Area proximal to the enlarged node

ANS: D When nodes are abnormal, the nurse should check the area into which they drain for the source of the problem. The area proximal (upstream) to the location of the abnormal node should be explored.

A patient visits the clinic because he has recently noticed that the left side of his mouth is paralyzed. He states that he cannot whistle but the nurse notes he can still raise his eyebrows. What does the nurse suspect? a. Bell palsy b. Cushing syndrome c. Parkinson syndrome d. Experienced a cerebrovascular accident (CVA) or stroke

ANS: D With an upper motor neuron lesion, as with a CVA, the patient will have paralysis of lower facial muscles, but the upper half of the face will not be affected owing to the intact nerve from the unaffected hemisphere. The person is still able to wrinkle the forehead and close the eyes. Bell palsy presents as complete paralysis of one side of the face. The person cannot wrinkle forehead, raise eyebrows, close eyelids, whistle, or show teeth on the affected side. With Cushing syndrome the person develops a rounded, "moonlike" face, prominent jowls, red cheeks, hirsutism on the upper lip, lower cheeks, and chin, and acneiform rash on the chest. The facial features characteristic of Parkinson syndrome are a flat and expressionless, "masklike," with elevated eyebrows, staring gaze, oily skin, and drooling. This patient who cannot whistle but can still raise his eyebrows has probably experienced a cerebrovascular accident.

A patient comes to the emergency department after a boxing match, and his O.S. is almost swollen shut. He has bruises on his face and neck. He says he is worried because he "can't see well" from his O.S. The physician suspects retinal detachment. What finding would support this suspicion? a. Loss of central vision b. Loss of peripheral vision c. Sudden loss of pupillary constriction and accommodation d. Shadow or diminished vision in one quadrant or one half of the visual field

ANS: D With retinal detachment, the person has shadows or diminished vision in one quadrant or one half of the visual field. The other responses are not signs of retinal detachment. Loss of central or peripheral vision or sudden loss of pupillary constriction and accommodation are not signs of retinal detachment.

During an examination, the nurse finds that a patient's left temporal artery is tortuous and feels hardened and tender, compared with the right temporal artery. The nurse suspects which condition? a. Bell palsy b. Crepitation c. Mastoiditis d. Temporal arteritis

ANS: D With temporal arteritis, the artery appears more tortuous and feels hardened and tender. These assessment findings are not consistent with the other responses. Bell palsy presents as complete paralysis of one side of the face. The person cannot wrinkle forehead, raise eyebrows, close eyelids, whistle, or show teeth on the affected side. Crepitation is a crackling sound. Mastoiditis is an inflammation of the mastoid process which is behind the ears. The signs and symptoms this patient has are consistent with temporal arteritis. With temporal arteritis, the artery appears more tortuous and feels hardened and tender.


Related study sets

HIGH RISK PREGNANCIES AND COMPLICATIONS

View Set

Carmen Homework 9 - Normal Distribution

View Set